Đến nội dung

Hình ảnh

Inequalities From 2016 Mathematical Olympiads

* * * * * 3 Bình chọn

  • Please log in to reply
Chủ đề này có 139 trả lời

#1
Nguyenhuyen_AG

Nguyenhuyen_AG

    Trung úy

  • Thành viên nổi bật 2016
  • 945 Bài viết

*
Phổ biến

Chào các bạn như tiêu đề đã ghi thì trong topic này chúng ta sẽ cùng thảo luận về các bài toán bất đẳng thức đến từ các cuộc thi học sinh giỏi Toán khắp nơi trên thế giới trong năm 2016 và kết thúc topic này (có thể là sau IMO 2016) chúng ta sẽ có một file pdf “Inequalities From 2016 Mathematical Olympiads” made in VMF để các thành viên có tài liệu tham khảo, hi vọng các bạn ủng hộ. :)

 

Bài 1 (Japan MO Preliminary). Với $a,\,b,\,c,\,d$ là bốn số thực thỏa mãn
\[\left\{\begin{aligned}&(a+b)(c+d)=2\\&(a+c)(b+d)=3\\&(a+d)(b+c)=4\end{aligned}\right.\]
Tìm giá trị nhỏ nhất của biểu thức $a^2+b^2+c^2+d^2.$

 

Bài 2 (Korea Winter Program Practice Test). Với số nguyên dương $n \geqslant 2$ và $a_i,\,b_i\;(1 \leqslant i \leqslant n)$ là các số thực dương thỏa mãn $\sum_{i=1}^n a_i = \sum_{i=1}^n b_i.$ Chứng minh rằng 

\[\sum_{i=1}^n \frac{(a_{i+1}+b_{i+1})^2}{n(a_i-b_i)^2+4(n-1)\displaystyle \sum_{j=1}^n a_jb_j} \geqslant \frac{1}{n-1}.\]

Bài 3 (Korea Winter Program Practice Test). Cho ba số thực không âm $x,\,y,\,z$ thỏa mãn

\[(x+y-1)^2+(y+z-1)^2+(z+x-1)^2=27.\]
Tìm giá trị lớn nhất và giá trị nhỏ nhất của biểu thức $x^4+y^4+z^4.$

 

Bài 4 (Romanian Masters In Mathematics). Cho hai số thực dương $x,\,y$ thỏa mãn $x+y^{2016}\geqslant 1.$ Chứng minh rằng $$x^{2016}+y> 1-\frac{1}{100}.$$
Bài 5 (San Diego Math Olympiad). Cho ba số thực dương $a,\,b,\,c$ thỏa mãn điều kiện $a\sqrt{bc}+b\sqrt{ca}+c\sqrt{ab} \geqslant 1.$ Tìm giá trị nhỏ nhất của $a+b+c.$

 

Bài 6 (Hong Kong TST). Cho ba số thực dương $a,\,b,\,c$ thỏa mãn $abc=1.$ Tìm giá trị nhỏ nhất của
$$\frac{a^3+8}{a^3(b+c)}+\frac{b^3+8}{b^3(a+c)}+\frac{c^3+8}{c^3(b+a)}.$$

Bài 7 (Hong Kong TST). Với $n$ là một số nguyên dương, giả sử tồn tại $n$ số thực dương $x_1,\,x_2,\,\ldots,\,x_n$ khác nhau sao cho mọi số nguyên $1 \leqslant i,\,j \leqslant n,$ thì $$(3x_i-x_j) (x_i-3x_j)\geqslant (1-x_ix_j)^2.$$ Tìm giá trị lớn nhất có thể có của $n.$
 

Bài 8 (Selection Of Kiev To UMO). Với $a,\,b,\,c$ là ba số thực dương thỏa mãn $a+b+c=3.$ Chứng minh rằng
\[\frac{a^2}{a+b^2}+\frac{b^2}{b+c^2}+\frac{c^2}{c+a^2} \geqslant \frac{3}{2}.\]

Bài 9 (Selection Of Kiev To UMO). Với $x,\,y,\,z$ là ba số thực dương. Chứng minh rằng $$\frac{x^2}{xy+z}+\frac{y^2}{yz+x}+\frac{z^2}{xz+y}\ge \frac{(x+y+z)^3}{3(x^2(y+1)+y^2(z+1)+z^2(x+1)}.$$

Bài 10 (CHKMO). Với $n$ số thực $a_1,\,a_2,\,\ldots,\,a_n$ thuộc $(1,-1)$ và số nguyên $1 \leqslant i \leqslant n$ thỏa mãn điều kiện

  • $a_1+a_2+\cdots+a_n=0.$
  • $a_1^2+a_2^2+\cdots+a_n^2=40.$

Tìm giá trị nhỏ nhất của $n.$

 

Bài 11 (Silk Road Mathematical Olympiad). Với $a,\,b,\,c$ là ba số thực thỏa mãn $| (a-b) (b-c) (c-a) | = 1.$ Tìm giá trị nhỏ nhất của $| a | + | b | + | c |.$
 

Bài 12 (China Junior High School Mathematics League). Với $x,\,y,\,z$ là ba số thực dương thỏa mãn $xy+yz+zx\neq 1$ và $$\frac{(x^2-1)(y^2-1)}{xy} +\frac{(y^2-1)(z^2-1)}{yz} +\frac{(z^2-1)(x^2-1)}{zx} =4.$$ Chứng minh rằng $9(x+y)(y+z)(z+x)\geqslant 8xyz(xy+yz+zx).$

 

Bài 13 (Final Korean Mathematical Olympiad). Cho ba số thực $x,y,z$ thỏa mãn điều kiện $x^2+y^2+z^2=1.$ Tìm giá trị lớn nhất của biểu thức \[P = (x^2-yz)(y^2-zx)(z^2-xy).\]

Bài 14 (Turkey TST). Với $a,\,b,\,c$ là ba số thực không âm thỏa mãn điều kiện $a^2+b^2+c^2 \leqslant 3.$ Chứng minh rằng
$$(a+b+c)(a+b+c-abc)\ge2(a^2b+b^2c+c^2a).$$

Bài 15 (Australien MO). Cho $a,b$ là hai số thực thỏa mãn $a^{2}+b^{2}=1.$ Chứng minh rằng
$$\left | a+\frac{a}{b}+b+\frac{b}{a} \right |\geq 2-\sqrt{2}.$$

Bài 16 (Israel Winter Camp). Với $a,\,b,\,c$ là ba số thực bất kỳ. Chứng minh rằng
\[4(a^6+b^6+c^6)+5abc(a^3+b^3+c^3)\geq(ab+ac+bc)^3.\]

 

Lưu ý. Các bài được tô màu xanh là các bài toán đã được giải.


Bài viết đã được chỉnh sửa nội dung bởi Nguyenhuyen_AG: 10-04-2016 - 12:18
sửa tiêu đề: thêm chữ Olympiads

Nguyen Van Huyen
Ho Chi Minh City University Of Transport

#2
Nguyenhuyen_AG

Nguyenhuyen_AG

    Trung úy

  • Thành viên nổi bật 2016
  • 945 Bài viết

Bài 17 (Azerbaijan Junior Mathematical Olympiad). Với $x,\,y,\,z$ là ba số thực khác $0.$ Chứng minh rằng $$\sqrt {x^2+\frac {1}{y^2}}+ \sqrt {y^2+\frac {1}{z^2}}+ \sqrt {z^2+\frac {1}{x^2}}\geq 3\sqrt {2}. $$

Bài 18 (Macedonia National Olympiad). Cho $n \;(n \geqslant 3)$ số thực dương $a_1,\,a_2,\,\ldots,a_n$ thỏa mãn điều kiện

$$\frac{1}{1+a_1^4} + \frac{1}{1+a_2^4} + \cdots + \frac{1}{1+a_n^4} = 1.$$

Chứng minh rằng $$a_1a_2 \cdots a_n \ge (n-1)^{\frac n4}.$$

Bài 19 (Cyprus TST). Giả sử $a,\,b,\,c$ là độ dài ba cạnh của tam giác $ABC$ và thỏa mãn điều kiện
$$a\sqrt{8}+b\sqrt{6}+c\sqrt{2}\ge 4\sqrt{a^2+b^2+c^2},$$
Chứng minh rằng $ABC$ là tam giác vuông.

 


Bài viết đã được chỉnh sửa nội dung bởi Nguyenhuyen_AG: 10-04-2016 - 12:17

Nguyen Van Huyen
Ho Chi Minh City University Of Transport

#3
Nguyenhuyen_AG

Nguyenhuyen_AG

    Trung úy

  • Thành viên nổi bật 2016
  • 945 Bài viết

Đang cập nhật ...


Nguyen Van Huyen
Ho Chi Minh City University Of Transport

#4
I Love MC

I Love MC

    Đại úy

  • Thành viên nổi bật 2016
  • 1861 Bài viết

Bài 6 (Hong Kong TST). Cho ba số thực dương $a,\,b,\,c$ thỏa mãn $abc=1.$ Tìm giá trị nhỏ nhất của

$$\frac{a^3+8}{a^3(b+c)}+\frac{b^3+8}{b^3(a+c)}+\frac{c^3+8}{c^3(b+a)}.$$

 

 

Không biết giải bài ở đây có vi phạm không anh nhỉ :( 
$VT=\sum \frac{a^3+1+1+6}{a^3(b+c)} \ge \sum \frac{3a+6}{a^3(b+c)}=\sum \frac{3(a+2)}{a^3(b+c)}$ 
Ta sẽ chứng minh $\sum \frac{3(a+2)}{a^3(b+c)} \ge \frac{27}{2}$ (*)
Chợt nhận thấy bài toán quen thuộc của IMO 1995  
Nếu $abc=1$ thì $\sum \frac{1}{a^3(b+c)} \ge \frac{3}{2}$ 
Áp dụng suy ra $\frac{6}{a^3(b+c)} \ge 9$ 
Lại có $\sum \frac{3}{a^2(b+c)}=\sum \frac{3(bc)^2}{b+c} \ge \frac{3(\sum ab)^2}{2\sum a}$ (1) 
Lại có $(\sum ab)^2 \ge 3.abc(a+b+c)$ nên từ (1) suy ra $\frac{3(\sum ab)^2}{2\sum a} \ge \frac{9}{2}$ 
Cộng lại suy ra (*) được chứng minh 
Vậy giá trị nhỏ nhất là $\frac{27}{2}$ khi và chỉ khi $a=b=c=1$



#5
Nguyenhuyen_AG

Nguyenhuyen_AG

    Trung úy

  • Thành viên nổi bật 2016
  • 945 Bài viết

Không biết giải bài ở đây có vi phạm không anh nhỉ :(

 

Đúng rồi em, mình giải ở đây luôn. :)


Nguyen Van Huyen
Ho Chi Minh City University Of Transport

#6
NTA1907

NTA1907

    Thượng úy

  • Thành viên
  • 1014 Bài viết

Bài 6 (Hong Kong TST). Cho ba số thực dương $a,\,b,\,c$ thỏa mãn $abc=1.$ Tìm giá trị nhỏ nhất của

$$\frac{a^3+8}{a^3(b+c)}+\frac{b^3+8}{b^3(a+c)}+\frac{c^3+8}{c^3(b+a)}.$$

 

Không biết giải bài ở đây có vi phạm không anh nhỉ :( 
$VT=\sum \frac{a^3+1+1+6}{a^3(b+c)} \ge \sum \frac{3a+6}{a^3(b+c)}=\sum \frac{3(a+2)}{a^3(b+c)}$ 
Ta sẽ chứng minh $\sum \frac{3(a+2)}{a^3(b+c)} \ge \frac{27}{2}$ (*)
Chợt nhận thấy bài toán quen thuộc của IMO 1995  
Nếu $abc=1$ thì $\sum \frac{1}{a^3(b+c)} \ge \frac{3}{2}$ 
Áp dụng suy ra $\frac{6}{a^3(b+c)} \ge 9$ 
Lại có $\sum \frac{3}{a^2(b+c)}=\sum \frac{3(bc)^2}{b+c} \ge \frac{3(\sum ab)^2}{2\sum a}$ (1) 
Lại có $(\sum ab)^2 \ge 3.abc(a+b+c)$ nên từ (1) suy ra $\frac{3(\sum ab)^2}{2\sum a} \ge \frac{9}{2}$ 
Cộng lại suy ra (*) được chứng minh 
Vậy giá trị nhỏ nhất là $\frac{27}{2}$ khi và chỉ khi $a=b=c=1$

Cách khác

Ta có:
$\sum \frac{a^{3}+8}{a^{3}(b+c)}=\sum \frac{1}{b+c}+\sum \frac{8}{a^{3}(b+c)}\geq \frac{9}{2(a+b+c)}+\sum \frac{8(bc)^{2}}{a(b+c)}\geq \frac{9}{2(a+b+c)}+\frac{8(bc+ca+ab)^{2}}{2(ab+bc+ca)}=\frac{9}{2(a+b+c)}+(ab+bc+ca)+3(ab+bc+ca)\geq \frac{9}{2(a+b+c)}+\sqrt{3abc(a+b+c)}+3.3\sqrt[3]{(abc)^{2}}=\frac{9}{2(a+b+c)}+\frac{\sqrt{3(a+b+c)}}{2}+\frac{\sqrt{3(a+b+c)}}{2}+9\geq 3\sqrt[3]{\frac{27}{8}}+9=\frac{27}{2}$
Dấu = xảy ra khi và chỉ khi $a=b=c=1$

Vũ trụ không có biên trong không gian, không có bắt đầu và kết thúc trong thời gian và chẳng có việc gì cho đấng sáng thế phải làm ở đây cả.

 


#7
dark templar

dark templar

    Kael-Invoker

  • Hiệp sỹ
  • 3788 Bài viết

Chào các bạn như tiêu đề đã ghi thì trong topic này chúng ta sẽ cùng thảo luận về các bài toán bất đẳng thức đến từ các cuộc thi học sinh giỏi Toán khắp nơi trên thế giới trong năm 2016 và kết thúc topic này (có thể là sau IMO 2016) chúng ta sẽ có một file pdf “Inequalities From 2016 Mathematical” made in VMF để các thành viên có tài liệu tham khảo, hi vọng các bạn ủng hộ. :)

 

Bài 2 (Korea Winter Program Practice Test). Với số nguyên dương $n \geqslant 2$ và $a_i,\,b_i\;(1 \leqslant i \leqslant n)$ là các số thực dương thỏa mãn $\sum_{i=1}^n a_i = \sum_{i=1}^n b_i.$ Chứng minh rằng 

\[\sum_{i=1}^n \frac{(a_{i+1}+b_{i+1})^2}{n(a_i-b_i)^2+4(n-1)\displaystyle \sum_{j=1}^n a_jb_j} \geqslant \frac{1}{n-1}.\]

 

 

Bạn Huyện cho mình hỏi là bài này có ký hiệu $a_{n+1}=a_1$ và $b_{n+1}=b_1$ không vậy ?


"Do you still... believe in me ?" Sarah Kerrigan asked Jim Raynor - Starcraft II:Heart Of The Swarm.

#8
NTA1907

NTA1907

    Thượng úy

  • Thành viên
  • 1014 Bài viết

Bài 8 (Selection Of Kiev To UMO). Với $a,\,b,\,c$ là ba số thực dương thỏa mãn $a+b+c=3.$ Chứng minh rằng

\[\frac{a^2}{a+b^2}+\frac{b^2}{b+c^2}+\frac{c^2}{c+a^2} \geqslant \frac{3}{2}.\]

Áp dụng AM-GM ta có:

$a-\frac{a^{2}}{a+b^{2}}=\frac{ab^{2}}{a+b^{2}}\leq \frac{ab^{2}}{2b\sqrt{a}}=\frac{b\sqrt{a}}{2}$

Tương tự cộng lại ta được:

$\sum \frac{a}{a+b^{2}}\geq 3-\frac{1}{2}(b\sqrt{a}+c\sqrt{b}+a\sqrt{c})\geq 3-\frac{1}{2}.\sqrt{(a+b+c)(ab+bc+ca)}\geq 3-\frac{1}{2}.\sqrt{(a+b+c).\frac{(a+b+c)^{2}}{3}}=3-\frac{3}{2}=\frac{3}{2}$

Dấu = xảy ra$\Leftrightarrow a=b=c=1$


Vũ trụ không có biên trong không gian, không có bắt đầu và kết thúc trong thời gian và chẳng có việc gì cho đấng sáng thế phải làm ở đây cả.

 


#9
trananhduong62

trananhduong62

    Hạ sĩ

  • Thành viên
  • 81 Bài viết
Anh vào chỗ nào để lấy các đề này thế

trananhduong62 :icon6:  :icon6:  :icon6:  :ukliam2: GOOD!


#10
Nguyenhuyen_AG

Nguyenhuyen_AG

    Trung úy

  • Thành viên nổi bật 2016
  • 945 Bài viết

Bạn Huyện cho mình hỏi là bài này có ký hiệu $a_{n+1}=a_1$ và $b_{n+1}=b_1$ không vậy ?

 

Đề gốc mình không thấy có ký hiệu này.


Nguyen Van Huyen
Ho Chi Minh City University Of Transport

#11
Gachdptrai12

Gachdptrai12

    Thượng sĩ

  • Điều hành viên THCS
  • 280 Bài viết

 

Bài 9 (Selection Of Kiev To UMO). Với $x,\,y,\,z$ là ba số thực dương. Chứng minh rằng $$\frac{x^2}{xy+z}+\frac{y^2}{yz+x}+\frac{z^2}{xz+y}\ge \frac{(x+y+z)^3}{3(x^2(y+1)+y^2(z+1)+z^2(x+1)}.$$

 

anh ơi bài ni đâu có đồng bậc 

p/s sửa xong xóa bl của em nha(nếu sai) :v



#12
trungvmfcsp

trungvmfcsp

    Binh nhì

  • Thành viên mới
  • 13 Bài viết

Em xin mạn phép viết lại suy nghĩ của e về bài 9 :|. Em thử làm hoy k biết có đúng hok, mong mọi người chỉ giáo :S

Áp dụng bđt Cauchy - Schwarz ta có:

$\frac{x^2}{xy+z}$ + $\frac{y^2}{yz+x}+\frac{z^2}{zx+y}$ $>= \frac{(x+y+z)^2}{x+y+z+xy+yz+zx}=\frac{(x+y+z)^3}{(x+y+z)(x+y+z+xy+yz+zx)}$

ĐPCM <=> $3(x^2(y+1)+y^2(z+1)+z^2(x+1)) >= (x+y+z)(x+y+z+xy+yz+zx)$

$<=>(x+y+z)^2+(x+y+z)(xy+yz+zx)<=3(x^2+y^2+z^2)+3(x^2y+y^2z+z^2x)$

$<=>x^2+y^2+z^2+2(xy+yz+zx)+xy(x+y)+yz(y+z)+zx(z+x)+3xyz<=3(x^2+y^2+z^2)+3(x^2y+y^2z+z^2x)$

$<=>2(xy+yz+zx)+xy^2+yz^2+zx^2+3xyz<=2(x^2+y^2+z^2+x^2y+y^2z+z^2x)$

$Do:xy+yz+zx <= x^2+y^2+z^2$

=> ĐPCM <=> $xy^2+yz^2+zx^2+3xyz<=2(x^2y+y^2z+z^2x)$

Mà: $x^2y+y^2z+z^2x>=3xyz$ (do AM-GM)

=> ĐPCM <=> $xy^2 + yz^2 + zx^2<=x^2y+y^2z+z^2x$

<=> $xy(x-y)+yz(y-z)+zx(z-x)>=0$ (*)

Không làm mất tính tổng quát, ta giả sử $x>=y>=z$

(*) <=>$xy(x-y)+yz(y-z)+zx(z-y+y-x)>=0$

<=> $xy(x-y)+yz(y-z)-zx(y-z)-zx(x-y)>=0$

<=>$x(x-y)(y-z)-z(x-y)(y-z)>=0$

<=>$(x-y)(y-z)(x-z)>=0$ (đúng)

=> ĐPCM


Bài viết đã được chỉnh sửa nội dung bởi trungvmfcsp: 04-04-2016 - 20:56


#13
trungvmfcsp

trungvmfcsp

    Binh nhì

  • Thành viên mới
  • 13 Bài viết

Em xin mạn phép viết lại suy nghĩ của e về bài 9 :|. Em thử làm hoy k biết có đúng hok, mong mọi người chỉ giáo :S

Áp dụng bđt Cauchy - Schwarz ta có:

$\frac{x^2}{xy+z}$ + $\frac{y^2}{yz+x}+\frac{z^2}{zx+y}$ $>= \frac{(x+y+z)^2}{x+y+z+xy+yz+zx}=\frac{(x+y+z)^3}{(x+y+z)(x+y+z+xy+yz+zx)}$

ĐPCM <=> $3(x^2(y+1)+y^2(z+1)+z^2(x+1)) >= (x+y+z)(x+y+z+xy+yz+zx)$

$<=>(x+y+z)^2+(x+y+z)(xy+yz+zx)<=3(x^2+y^2+z^2)+3(x^2y+y^2z+z^2x)$

$<=>x^2+y^2+z^2+2(xy+yz+zx)+xy(x+y)+yz(y+z)+zx(z+x)+3xyz<=3(x^2+y^2+z^2)+3(x^2y+y^2z+z^2x)$

$<=>2(xy+yz+zx)+xy^2+yz^2+zx^2+3xyz<=2(x^2+y^2+z^2+x^2y+y^2z+z^2x)$

$Do:xy+yz+zx <= x^2+y^2+z^2$

=> ĐPCM <=> $xy^2+yz^2+zx^2+3xyz<=2(x^2y+y^2z+z^2x)$

Mà: $x^2y+y^2z+z^2x>=3xyz$ (do AM-GM)

=> ĐPCM <=> $xy^2 + yz^2 + zx^2<=x^2y+y^2z+z^2x$

<=> $xy(x-y)+yz(y-z)+zx(z-x)>=0$ (*)

Không làm mất tính tổng quát, ta giả sử $x>=y>=z$

(*) <=>$xy(x-y)+yz(y-z)+zx(z-y+y-x)>=0$

<=> $xy(x-y)+yz(y-z)-zx(y-z)-zx(x-y)>=0$

<=>$x(x-y)(y-z)-z(x-y)(y-z)>=0$

<=>$(x-y)(y-z)(x-z)>=0$ (đúng)

=> ĐPCM



#14
Gachdptrai12

Gachdptrai12

    Thượng sĩ

  • Điều hành viên THCS
  • 280 Bài viết
1 cách khác cho bài 9 (đề đúng có lẽ em nhầm) Đặt bđt là (1)
áp dụng bđt holder ta có
(1+1+1)(x(xy+z)+y(yz+x)+z(zx+x))(VT(1))>=(x+y+z)^3
nên để chungs minh (1) ta chứng minh
x(xy+z)+y(yz+x)+z(zx+y)<=x^2(y+1)+y^2(y+1)+z^2(x+1) khai triển ra ta thấy bđt trên <=> x^2+y^2+z^2>= xy+yz+zx đúng dpcm
p.s nhờ anh huyện sửa bài em thành latex nha em dùng dtdd nên ko viết latex dc

Bài viết đã được chỉnh sửa nội dung bởi Gachdptrai12: 04-04-2016 - 21:25


#15
Gachdptrai12

Gachdptrai12

    Thượng sĩ

  • Điều hành viên THCS
  • 280 Bài viết
13)(koreanMO2016 ngày2 )cho $x,y,z$ là số thực thỏa $x^2+y^2+z^2=1$, tìm max $$(x^2-yz)(y^2-zx)(z^2-xy)$$

Bài viết đã được chỉnh sửa nội dung bởi Gachdptrai12: 05-04-2016 - 09:18


#16
tungteng532000

tungteng532000

    Trung sĩ

  • Thành viên
  • 174 Bài viết

Bài 5 (San Diego Math Olympiad). Cho ba số thực dương a,b,ca,b,c thỏa mãn điều kiện abc+bca+cab1. Tìm giá trị nhỏ nhất của a+b+c.

 

 

Từ giả thiết, áp dụng bđt AM-GM, ta được:
$1\leq \sqrt{abc}(\sqrt{a}+\sqrt{b}+\sqrt{c})\leq \frac{1}{9}(\sqrt{a}+\sqrt{b}+\sqrt{c})(a+b+c)(\sqrt{a}+\sqrt{b}+\sqrt{c})\leq \frac{1}{3}(a+b+c)^2\Rightarrow a+b+c\geq \sqrt{3}$
Đẳng thức xảy ra khi a=b=c=$\frac{1}{\sqrt{3}}$


Bài viết đã được chỉnh sửa nội dung bởi tungteng532000: 04-04-2016 - 22:09

                                              Lời giải hay thì like nhé :))
FB: 
https://www.facebook...oylanh.lung.564


#17
Ankh

Ankh

    Hạ sĩ

  • Thành viên
  • 85 Bài viết

 Bài 11. Không giảm tính tổng quát giả sử $a>b>c$ ( hiển nhiên $a,\ b,\ c$ đôi một không đồng thời bằng nhau )

 Khi đó ta có $1=(a-b)(b-c)(a-c)\leq \dfrac{1}{4}(a-c)^3\Rightarrow a-c\geq \sqrt[3]{4}$

 Lại có $|a|+|b|+|c|\geq |a-c|+|b|\geq a-c\geq \sqrt[3]{4}$ nên ta có giá trị nhỏ nhất

 Dấu "=" xảy ra khi và chỉ khi $b=0;a=-c=\dfrac{1}{\sqrt[3]{2}}$

 Mà mình thấy có mấy câu hơi dở đi nhỉ  :wacko:



#18
Ankh

Ankh

    Hạ sĩ

  • Thành viên
  • 85 Bài viết

 Bài 2. Ta chú ý đến một phân tích quen thuộc là $(x+y)^2=(x-y)^2+4xy$

 Bây giờ đặt $\sum _{j=1}^na_jb_j=S$ và $a_{i}b_{i}=S_i$ thì ta cần chứng minh $\sum_{i=1}^n \dfrac{(a_{i+1}+b_{i+1})^2}{n(a_i-b_i)^2+4(n-1)S} \geq \dfrac{1}{n-1}$

 Hay $\sum_{i=1}^n \dfrac{(a_{i+1}-b_{i+1})^2+4S_{i+1}}{n(a_i-b_i)^2+4(n-1)S} \geq \dfrac{1}{n-1}$

 Không mất tính tổng quát giả sử $(a_1-b_1)^2\leq \min \left \{(a_2-b_2)^2,(a_3-b_3)^2,...,(a_n-b_n)^2\right \}$ thì ta có

 $\sum_{i=1}^n \dfrac{(a_{i+1}-b_{i+1})^2+4S_{i+1}}{n(a_i-b_i)^2+4(n-1)S} \geq \dfrac{\sum \limits_{i=1}^n (a_{i}-b_{i})^2+4\sum \limits_{k=1}^nS_{k}}{n(a_1-b_1)^2+4(n-1)S}=\dfrac{\sum \limits_{i=1}^n (a_{i}-b_{i})^2+4S}{n(a_1-b_1)^2+4(n-1)S}$

 Nên ta chỉ cần chứng minh $\dfrac{\sum \limits_{i=1}^n (a_{i}-b_{i})^2+4S}{n(a_1-b_1)^2+4(n-1)S}\geq \dfrac{1}{n-1}\Leftrightarrow \sum_{i=1}^n (a_{i}-b_{i})^2\geq \dfrac{n}{n-1}(a_1-b_1)^2\Leftrightarrow \sum_{i=2}^n (a_{i}-b_{i})^2\geq \dfrac{1}{n-1}\geq (a_1-b_1)^2$

 Áp dụng bất đẳng thức Cauchy-Schwarz ta có $\sum_{i=2}^n (a_{i}-b_{i})^2\geq \dfrac{1}{n-1}\left (\sum_{j=2}^n a_j-\sum_{k=2}^n b_k \right )^2$

 Mặt khác $\sum_{i=1}^n (a_i-b_i)=0$ nên $\left (\sum_{j=2}^n a_j-\sum_{k=2}^n b_k \right )^2=(a_1-b_1)^2$

 Từ đó ta có điều cần chứng minh ~.~ Mỏi tay


Bài viết đã được chỉnh sửa nội dung bởi Ankh: 05-04-2016 - 00:32


#19
trungvmfcsp

trungvmfcsp

    Binh nhì

  • Thành viên mới
  • 13 Bài viết

Em xin mạn phép viết lại suy nghĩ của e về bài 9 :|. Em thử làm hoy k biết có đúng hok, mong mọi người chỉ giáo :S

Áp dụng bđt Cauchy - Schwarz ta có:

$\frac{x^2}{xy+z}$ + $\frac{y^2}{yz+x}+\frac{z^2}{zx+y}$ $>= \frac{(x+y+z)^2}{x+y+z+xy+yz+zx}=\frac{(x+y+z)^3}{(x+y+z)(x+y+z+xy+yz+zx)}$

ĐPCM <=> $3(x^2(y+1)+y^2(z+1)+z^2(x+1)) >= (x+y+z)(x+y+z+xy+yz+zx)$

$<=>(x+y+z)^2+(x+y+z)(xy+yz+zx)<=3(x^2+y^2+z^2)+3(x^2y+y^2z+z^2x)$

$<=>x^2+y^2+z^2+2(xy+yz+zx)+xy(x+y)+yz(y+z)+zx(z+x)+3xyz<=3(x^2+y^2+z^2)+3(x^2y+y^2z+z^2x)$

$<=>2(xy+yz+zx)+xy^2+yz^2+zx^2+3xyz<=2(x^2+y^2+z^2+x^2y+y^2z+z^2x)$

$Do:xy+yz+zx <= x^2+y^2+z^2$

=> ĐPCM <=> $xy^2+yz^2+zx^2+3xyz<=2(x^2y+y^2z+z^2x)$

Mà: $x^2y+y^2z+z^2x>=3xyz$ (do AM-GM)

=> ĐPCM <=> $xy^2 + yz^2 + zx^2<=x^2y+y^2z+z^2x$

<=> $xy(x-y)+yz(y-z)+zx(z-x)>=0$ (*)

Không làm mất tính tổng quát, ta giả sử $x>=y>=z$

(*) <=>$xy(x-y)+yz(y-z)+zx(z-y+y-x)>=0$

<=> $xy(x-y)+yz(y-z)-zx(y-z)-zx(x-y)>=0$

<=>$x(x-y)(y-z)-z(x-y)(y-z)>=0$

<=>$(x-y)(y-z)(x-z)>=0$ (đúng)

=> ĐPCM

Cơ mà hình như e chưa đúng, tại vì bđt này hoán vị nên ko gỉa sử dc chăng :? Cách xài Holder của bạn Gachdptrai12 đúng hơn đó ak :3 Mn tham khảo :?


Bài viết đã được chỉnh sửa nội dung bởi trungvmfcsp: 05-04-2016 - 00:55


#20
dark templar

dark templar

    Kael-Invoker

  • Hiệp sỹ
  • 3788 Bài viết

 Bài 2. Ta chú ý đến một phân tích quen thuộc là $(x+y)^2=(x-y)^2+4xy$

 Bây giờ đặt $\sum _{j=1}^na_jb_j=S$ và $a_{i}b_{i}=S_i$ thì ta cần chứng minh $\sum_{i=1}^n \dfrac{(a_{i+1}+b_{i+1})^2}{n(a_i-b_i)^2+4(n-1)S} \geq \dfrac{1}{n-1}$

 Hay $\sum_{i=1}^n \dfrac{(a_{i+1}-b_{i+1})^2+4S_{i+1}}{n(a_i-b_i)^2+4(n-1)S} \geq \dfrac{1}{n-1}$

 Không mất tính tổng quát giả sử $(a_1-b_1)^2\leq \min \left \{(a_2-b_2)^2,(a_3-b_3)^2,...,(a_n-b_n)^2\right \}$ thì ta có

 $\sum_{i=1}^n \dfrac{(a_{i+1}-b_{i+1})^2+4S_{i+1}}{n(a_i-b_i)^2+4(n-1)S} \geq \dfrac{\sum \limits_{i=1}^n (a_{i}-b_{i})^2+4\sum \limits_{k=1}^nS_{k}}{n(a_1-b_1)^2+4(n-1)S }(1)=\dfrac{\sum \limits_{i=1}^n (a_{i}-b_{i})^2+4S}{n(a_1-b_1)^2+4(n-1)S}$

 Nên ta chỉ cần chứng minh $\dfrac{\sum \limits_{i=1}^n (a_{i}-b_{i})^2+4S}{n(a_1-b_1)^2+4(n-1)S}\geq \dfrac{1}{n-1}\Leftrightarrow \sum_{i=1}^n (a_{i}-b_{i})^2\geq \dfrac{n}{n-1}(a_1-b_1)^2\Leftrightarrow \sum_{i=2}^n (a_{i}-b_{i})^2\geq \dfrac{1}{n-1}\geq (a_1-b_1)^2$

 Áp dụng bất đẳng thức Cauchy-Schwarz ta có $\sum_{i=2}^n (a_{i}-b_{i})^2\geq \dfrac{1}{n-1}\left (\sum_{j=2}^n a_j-\sum_{k=2}^n b_k \right )^2 (2)$

 Mặt khác $\sum_{i=1}^n (a_i-b_i)=0$ nên $\left (\sum_{j=2}^n a_j-\sum_{k=2}^n b_k \right )^2=(a_1-b_1)^2$

 Từ đó ta có điều cần chứng minh ~.~ Mỏi tay

Chỗ khúc $(1)$ thì tử số của mẫu chỉ là $\sum_{k=1}^{n-1}\left ( a_{k}-b_{k} \right )^{2}+\left ( a_{n+1}-b_{n+1} \right )^{2}+\sum_{k=2}^{n}S_{k}+b_{n+1}a_{n+1}$.Đó là lý do mình hỏi có cái quy ước nào cho $a_{n+1}$ và $b_{n+1}$ hay không  :mellow:

 

Còn khúc $(2)$ thì nên viết rõ là $\sum_{i=2}^{n}\left ( a_{i}-b_{i} \right )^{2}\geqslant \frac{1}{n-1}\left ( \sum_{i=2}^{n}\left | a_{i}-b_{i} \right | \right )\geqslant \frac{1}{n-1}\left ( \sum_{k=2}^{n}a_{k}-\sum_{j=2}^{n}b_{j} \right )$


Bài viết đã được chỉnh sửa nội dung bởi dark templar: 05-04-2016 - 08:30

"Do you still... believe in me ?" Sarah Kerrigan asked Jim Raynor - Starcraft II:Heart Of The Swarm.




3 người đang xem chủ đề

0 thành viên, 3 khách, 0 thành viên ẩn danh